Download as docx, pdf, or txt
Download as docx, pdf, or txt
You are on page 1of 90

HBP:

Acute cholecystitis and septic shock on multiple pressors 


A
Laparoscopic cholecystectomy
B
Percutaneous cholecystostomy
C
Both
D
Neither
B
Percutaneous cholecystostomy

Acalculous cholecystitis
C
Both
Asymptomatic gall bladder polyp 5 mm in diameter without stones or signs
of malignancy
D
Neither
Acute cholecystitis and American Society of Anesthesiologists class III
A
Laparoscopic cholecystectomy

The Acute Cholecystectomy-Delayed Cholecystectomy study demonstrated


that for patients who were:
- not pregnant,
- not terminal,
- American Society of Anesthesiologists (ASA) class I–III,
- not in septic shock, and
- without a perforated gallbladder,
laparoscopic cholecystectomy within 24 hours for acute calculous
cholecystitis had significantly lower morbidity and cost.
Only 2 prospective trials have been conducted on percutaneous
cholecystostomy in high surgical risk acute calculous cholecystitis.

Both showed no significant difference in mortality and possibly reduced costs


and morbidity.

However, percutaneous cholecystostomy can be performed safely on


patients who are not otherwise operative candidates, such as :
- ASA class IV and V and
- patients with severe sepsis.

Percutaneous cholecystostomy may be the only treatment required for


acalculous cholecystitis in the critically ill, with success rates between 57%
and 100%.

However, successful laparoscopic cholecystectomy is reported in surgical


candidates.

Gallbladder polyps that are 6–9 mm on ultrasound without signs of


malignancy or risk factors for gallbladder cancer (e.g.,
- age over 60,
- sessile morphology,
- gallstones,
- primary sclerosing cholangitis)
can be safely observed with interval ultrasound studies.
===============================================
5-cm focal nodular hyperplasia (FNH), asymptomatic
A
Enucleation
B
Hypertonic saline injection
C
Observation
D
Resection to negative margins
E
Transarterial embolization

C
Observation

5-cm hepatic adenoma, asymptomatic

D
Resection to negative margins

5-cm hepatic peripheral hemangioma with abdominal pain

A
Enucleation

5-cm hepatic adenoma with active hemorrhage


E
Transarterial embolization

Incidentally discovered liver lesions are becoming increasingly common as


medical imaging continues to improve.
The majority of these lesions are benign and require no further treatment.
Treatment decisions are based on the
- size of the lesion as well as
- symptoms.

Hepatic hemangiomas are the most common benign neoplasm of the liver.

Hemangiomas have
- no risk of malignant degeneration and a
- very low risk of bleeding.
-
Lesions smaller than 5 cm rarely cause symptoms.
An asymptomatic hemangioma can be safely observed.

Hemangiomas that bleed may be embolized.

Indications for surgery are


- symptoms (recurrent abdominal pain)
- extremely large lesions (>10 cm).
o For those requiring surgery, resection is the treatment of choice.
Enucleation is possible in many cases, because the hemangioma tends to
compress adjacent liver tissue, creating a plane of dissection.

Focal nodular hyperplasia (FNH) is the second most common benign hepatic
lesion.

FNH has no potential for


- malignancy,
- spontaneous rupture, or
- hemorrhage.

The only indications for resection of FNH are


- symptoms (abdominal pain) and
- inability to rule out malignancy.
A definitive diagnosis, however, is usually possible with modern imaging
techniques.

Asymptomatic FNH does not require intervention.


Annual ultrasound for 2–3 years is prudent for women who wish to continue
oral contraceptive (OCP) use.

Hepatic adenomas (HA) are benign liver masses that tend to be hormonally
sensitive.

The classic presentation is in young women (aged 20s–40s) with a history of


OCP use and in men with a history of anabolic steroid use.

Many HAs regress with discontinuation of OCPs and steroids.

Management strategies of HA are more aggressive than for other benign


liver masses, because they have a risk of
- hemorrhage and
- malignant degeneration to hepatocellular carcinoma (HCC).

The risk of hemorrhage or an occult focus of HCC increases in HAs greater


than 5 cm in size.

Indications for surgery include


(1) size at least 5 cm,
(2) male patient, and
(3) inability to rule out malignancy.

Management of an acute rupture with hemorrhage is initial


angioembolization followed by elective resection.

If possible, resection to negative margins should be done in case underlying


HCC is identified on final pathologic analysis.

Hypertonic saline injections are used to treat hydatid disease as a


component of PAIR: percutaneous aspiration, injection of scolicidal agent,
and re-aspiration.

Presence of 2 hyperplastic polyps on initial screening colonoscopy


A
Repeat colonoscopy in 10 years
B
Repeat colonoscopy in 3 years
C
Repeat colonoscopy in 6 months
D
Repeat colonoscopy in 1 year
E
CT colonography in 6 months

A
Repeat colonoscopy in 10 years

Presence of 2 tubular adenomas, each less than 1 cm and without


dysplasia, on initial screening colonoscopy

A
Repeat colonoscopy in 10 years

Presence of 1.1-cm tubular adenoma with dysplasia, margin of resection


negative on initial screening colonoscopy

B
Repeat colonoscopy in 3 years

Presence of 0.5-cm tubulovillous adenoma without dysplasia, margin of


resection negative on initial screening colonoscopy

B
Repeat colonoscopy in 3 years
Guidelines for postpolypectomy colonoscopy surveillance are established to
minimize the risks of screening colonoscopy while preventing interval
development of cancers and cancer-related mortality.

Up to 17% of polyps at least 10 mm in size are missed during colonoscopy.

In many situations, subsequent cancer development may be more likely


related to these missed lesions than to interval development of new lesions
between colonoscopies.

Recommendations for repeat screening colonoscopy intervals are based on


the
- size and
- pathology
of polyp(s) removed at the index colonoscopy.

Hyperplastic polyps do not portend any increased risk of developing


colorectal malignancy and therefore require no increased surveillance from
the standard 10-year follow-up.

Patients with 1 or 2 adenomatous polyps, less than 10 mm in diameter, are


considered low risk, and 10-year follow-up is recommended.

Patients with 3 or more adenomatous polyps, 1 or more polyps greater than


10 mm in size, or adenomatous polyps with villous or high-grade histology
are considered high risk, and 3-year follow-up is recommended.

Six-month and 1-year follow-up regimens are not routinely recommended


for repeat screening colonoscopy or colonography intervals when polyps are
discovered.

Which of the following statements regarding the nonoperative management


of uncomplicated acute appendicitis is true?
A
No randomized trials compare nonoperative versus operative management.
B
Most failures of nonoperative management occur within the first year after
treatment.
C
Patients treated with surgery have fewer complications than patients treated
nonoperatively.
D
Nonoperative management is associated with increased mortality.
E
Failure of initial nonoperative management is associated with an increased
risk of developing complicated appendicitis.

B
Most failures of nonoperative management occur within the first year after
treatment.

Appendicitis is the most frequent intra-abdominal emergency, and surgery


has long been the mainstay of therapy.

Recently, multiple randomized clinical trials were conducted comparing


nonoperative management with antibiotics to surgery in patients with
uncomplicated appendicitis.

Nonoperative management, had a success rate of 44–85% in individual


studies.
Most recurrences in the nonoperative group occur within the first year after
treatment.

Surgery is associated with a higher rate of complications compared with


nonoperative management.

Mortality was similar between operative and nonoperative management.

A meta-analysis of 4 randomized trials consisting of 900 patients found


a relative risk reduction in complications of 31% in the antibiotic
treatment group compared with the appendectomy group.

Antibiotic therapy may delay appendectomy in patients who are not


improving, but this delay is not associated with an increased risk of
developing complicated appendicitis, including perforation or peritonitis.

Which of the following statements is true regarding the diagnosis and


management of hepatocellular carcinoma (HCC)?
A
PET scan is the preferred surveillance imaging in high-risk populations.
B
Alpha-fetoprotein greater than 3 times normal is diagnostic.
C
A tissue diagnosis is required before initiating therapy.
D
Screening reduces the overall mortality associated with HCC.
E
Triple-phase CT is diagnostic in high-risk populations.

E
Triple-phase CT is diagnostic in high-risk populations.

Hepatocellular carcinoma (HCC) is the fifth most common cancer in men and
ninth in women in the world.

It arises most commonly in the setting of chronic liver disease—cirrhosis


caused by hepatitis C virus, alcohol, and nonalcoholic steatohepatitis (NASH)
—although it is also seen before the onset of cirrhosis in the setting of
hepatitis B and sporadically in individuals older than 60 and in those with
certain types of hepatic adenoma (those with nuclear localization of beta-
catenin).

In patients with cirrhosis, the diagnosis of HCC can be made with triple-
phase CT scan.

Liver Imaging Reporting and Data System (LiRADS) criteria include


- the presence of arterial-phase enhancement (hypervascularity),
- portal venous phase washout, and
- threshold growth if a historical comparison is available.

The scale runs from 1 (definitely benign) to 5 (diagnostic of HCC).


LiRADS 4 and 5 lesions can be treated without a biopsy.

A tissue diagnosis may be necessary in low-risk patients and when there is


nondiagnostic imagining.
Serum alpha fetoprotein has a high positive predictive value when greater
than 500 ng/mL but a low negative predictive value.

Approximately 20% of patients with HCC have a normal alpha fetoprotein.

Alpha fetoprotein levels are not used as a diagnostic criterion for HCC.

An effective screening approach that significantly reduces HCC-related


mortality has not been identified.

Serial serum alpha fetoprotein screening in more than 5000 chronic hepatitis
B carriers in China identified more early-stage cancers than the unscreened
population but did not significantly reduce mortality.

The authors concluded this was likely a function of a lead-time bias—


diagnosing the disease earlier but not changing the disease-specific
mortality.

PET scans are neither sensitive nor specific for the diagnosis of HCC.

A patient with a cystic mass in the head of the pancreas undergoes cross-


sectional imaging and cystic fluid sampling. Which of the following clinical
presentations of a cystic mass in the pancreas has the highest risk of
malignancy?
 
A
Patient: 70-year-old woman
Size: 8 cm
Contents: Serous fluid
Main duct width: 1 cm
 
B
Patient: 80-year-old man
Size: 1.5 cm
Contents: Mucinous fluid
Main duct width: 0.4 cm
 
C
Patient: 65-year-old man
Size: 5 cm
Contents: Mucin
Main duct width: 1.5 cm
 
D
Patient: 32-year-old man with a recent history of acute pancreatitis
Size: 6 cm
Contents: Serous fluid
Main duct width: 0.8 cm
 
E
Patient: 55-year-old woman
Size: 1.5 cm
Contents: Mucin
Main duct width: 0.5 cm

C
Patient: 65-year-old man
Size: 5 cm
Contents: Mucin
Main duct width: 1.5 cm

Pancreatic cysts are more frequently recognized on cross-sectional


abdominal imaging than in the past.

A small percentage of pancreatic cysts can develop into pancreatic cancer,


and these patients may benefit from surgical resection.

The International Association of Pancreatology updated its consensus


guidelines for the surgical management of pancreatic cysts in 2012.

The updated guidelines attempted to be more accurate in predicting


malignant cysts, but they still have a relatively low positive predictive value.

The use of cyst fluid biopsy samples and other biomarkers to risk stratify
patients has not been successful in identifying patients whose cysts require
surgical resection.
Determinants of the malignant potential of a pancreatic cyst include:
- mucin in the cyst contents,
- cyst size,
- solid tumor in the cyst, and
- main pancreatic duct dilation.

Loculation within the cyst does not predict malignancy.

Cysts containing serous fluid are generally benign and include serous
cystadenomas and benign epithelial cysts.

Accordingly, the patient with the largest mucin-containing cyst and greatest
dilation of the main duct represents the patient with the highest risk of
associated malignancy.

Mucin-containing cysts include:


- intraductal papillary neoplasms (IPMN) and
- mucinous cystic tumors (MCN).
MCN occur almost exclusively in women 55–65 years old, w
IPMN occur in older men and women, with men predominating.

Current recommendation from guidelines recommend resection of:


- all MCN,
- main duct IPMN,
- branch duct-IPMN with a solid component,
- main pancreatic duct size at least 1 cm,
- obstructive jaundice, or
- cytology suspicious or positive for cancer.

A 56-year-old man with Child-Pugh B cirrhosis with hepatitis C is found to


have a 6-cm mass in segments II and III with arterial enhancement and
venous washout on triple-phase CT. His alpha-fetoprotein level is 450
ng/mL. An indocyanine green clearance (ICG) is obtained to assess his
suitability for resection. The minimal percentage clearance at 15 minutes
after injection of ICG needed to proceed with surgery is
A
50%.
B
60%.
C
70%.
D
80%.
E
90%.

D
80%

Hepatocellular carcinoma (HCC) is the most common primary malignancy of


the liver.
More than 80% of the time, it develops in the setting of cirrhosis resulting
from chronic liver disease due to etiologies such as chronic viral infection
(i.e., hepatitis B or C), alcoholism, and nonalcoholic steatohepatitis (NASH).

Other risk factors include exposure to substances such as aflatoxins, azo


dyes, aromatic amines, N-nitroso compounds, chlorinated hydrocarbons,
hydrosol compounds, pesticides, and radiation.

In addition, anabolic steroid and oral contraceptive use are risk factors.

In patients at risk for HCC, an alpha-fetoprotein level greater than 400


ng/mL is highly suspicious for the presence of HCC.

Diagnosis is made based on radiographic imaging criteria and clinical


findings.
On triphasic hepatic CT, HCC exhibits a classic arterial enhancement with
venous washout. 

Although only 30% of patients with HCC may qualify for operative
treatment, surgery remains the best option available.

Because the leading cause of death after resection for HCC is liver failure,
proper preoperative assessment of hepatic reserve is essential to identify
patients at risk.

Both the Child-Pugh and Model for End-Stage Liver Disease classifications
can aid in determining those patients at greatest risk.
For example, Child-Pugh C patients have a greater than 25% perioperative
mortality, and, consequently, resection is contraindicated. 

Triphasic liver CT can be used to calculate the future liver remnant.

A future liver remnant less than 40% is predictive of postoperative liver


failure.

Finally, indocyanine green (ICG) clearance can be used to assess the


functional capacity of the liver.

The dye is injected into the bloodstream, and clearance from the liver is
measured 15 minutes after injection.
Less than 10% (90% cleared) of the dye should be detectable at this point in
normal functioning livers.
With 15–20% retention,( 80% cleared) a 2-segment resection is
possible.
With 21–30% retention, (70% cleared) a single segment or wedge
resection is indicated.
Greater than 40% retention of dye at 15 minutes is predictive of
postoperative liver failure regardless of resection size.

In this patient requiring a 2-segment resection for treatment of his 6-cm


HCC, the minimum ICG clearance would be 80%, corresponding to a 20%
retention rate, which would allow performance of a 2-segment resection
without the development of liver failure.

A 58-year-old alcoholic man undergoes screening colonoscopy and is found


to have a large sessile lesion in his sigmoid colon positive for
adenocarcinoma on biopsy. On CT, he has a steatotic liver with synchronous
metastases throughout his right liver lobe and segments IVa and IVb of his
left lobe. The next step in his management should include 
A
chemotherapy alone.
B
diverting colostomy.
C
sigmoid colectomy.
D
extended right hepatectomy.
E
sigmoid colectomy with extended right hepatectomy.

A
chemotherapy alone.

Colorectal cancer is the second leading cause of cancer deaths worldwide,


and hepatic metastases accounts for two-thirds of these deaths.

Approximately 25% of patients with colorectal cancer present with


synchronous liver lesions, and 30% will develop metachronous liver
metastases during the course of their disease.

Resection remains the standard therapy for cure in patients with hepatic
involvement, with morbidity and mortality less than 30% and 3%,
respectively.

Patients who present with synchronous colorectal and hepatic lesions


present a challenge to surgeons.

Options for care include colon-first, liver-first, and concomitant resections of


disease in patients with resectable tumors and metastases.

In a recent systematic review of 18 papers on the topic, none of the 3


surgical strategies demonstrated inferiority compared with the others.
Hence, the approach taken should be based on patient factors.

An expert consensus came to the same conclusion, stating that resection


should be based on concern for
- complications at the primary tumor site,
- progression of liver disease, and
- complexity of simultaneous resection.

In patients with unresectable liver metastases, resection of the primary


tumor is no longer advocated in the absence of complications (i.e.,
obstruction, bleeding, perforation).

This evolution has occurred because of the introduction of more effective


chemotherapeutic drugs, such as oxaliplatin and irinotecan, to treat
colorectal cancer.
Consequently, primary chemotherapy leads to higher response and control
rates, resulting in complication rates dropping to less than 10%. 

In this patient with sigmoid cancer, hepatic metastases involving 80% of his
liver, and liver disease, chemotherapy alone is the most appropriate next
step in his management.

He is asymptomatic, making a diverting colostomy or sigmoid resection


unnecessary.

Given his liver disease and the extent of metastatic involvement, he would
not tolerate an extended right hepatectomy.
Finally, simultaneous resection is not indicated, because his liver disease is
unresectable.

A 63-year-old woman presents with acute onset of epigastric abdominal pain


and is found to have leukocytosis. Her abdomen is tender, but she does not
have peritonitis. Her CT shows a perforation of a duodenal diverticulum that
is contained. Which of the following statements is true?
A
A Whipple procedure is required.
B
Endoscopy is required.
C
Elevated serum amylase and lipase will confirm the diagnosis.
D
These diverticula commonly arise from the second portion of the duodenum.
E
Life-long proton pump inhibitor therapy will be required.

D
These diverticula commonly arise from the second portion of the duodenum.

The duodenum is the second most common site of diverticula formation in


the intestinal tract.
Duodenal diverticula are classified as either congenital or acquired.
Congenital duodenal diverticula contain all layers of the duodenal wall.

Acquired duodenal diverticula are pulsion diverticula containing only mucosa,


submucosa, and muscularis mucosa protruding through a weakness in the
duodenal wall.

The ampulla of Vater is an example of such an area of weakness in the


duodenal wall, accounting for the common location of diverticula within the
second portion.

Complications of duodenal diverticula, although rare, can include


- perforation,
- inflammation,
- compression of surrounding structures, or
- neoplastic transformation.

Surgical treatment options for perforation, especially in the presence of


generalized peritonitis or contamination, include
- surgical drainage,
- diverticulectomy, or
- pancreaticoduodenectomy.

In carefully selected cases of contained perforation, management can be


nonoperative with nasogastric tube decompression, antibiotics, and bowel
rest with or without endoscopically placed drains or stents.

The patient described has acute abdominal pain but without diffuse
peritonitis; her images show a contained perforation of the duodenum.
In this setting, nasogastric decompression, intravenous antibiotics, and
bowel rest are appropriate.
This is unrelated to peptic ulcer disease; therefore, proton pump inhibitor
therapy is not indicated.
Although amylase and lipase may be elevated, they are not diagnostic of
duodenal diverticula. Endoscopy and Whipple are not required in this setting.

A 55-year-old alcoholic man presents to clinic with signs and symptoms


consistent with chronic pancreatitis. CT reveals enlargement of the
pancreatic head with multiple stones and strictures in his proximal main
pancreatic duct. In discussing treatment options with him, which of the
following would result in the best long-term pain control?
A
Endoscopic sphincterotomy and stone extraction alone
B
Endoscopic sphincterotomy, stone extraction, and stent placement
C
Longitudinal pancreaticojejunostomy
D
Longitudinal pancreaticojejunostomy with limited resection head of pancreas
E
Pancreaticoduodenectomy with reconstruction

D
Longitudinal pancreaticojejunostomy with limited resection head of pancreas

Chronic pancreatitis is characterized by recurring inflammation in the


pancreas, which results in progressive fibrosis and manifests clinically as
pain, exocrine or endocrine dysfunction, and weight loss.

Approximately 80% of chronic pancreatitis is caused by excessive alcohol


consumption; other etiologies include autoimmune disorders,
choledocholithiasis, and congenital malformations.

Pharmacologic treatment of chronic pancreatitis consists of pain control and


replacement of exocrine and endocrine function as required.

Palliative options for long-term chronic pancreatitis-induced pain include


procedures involving:
- pancreatic head resections, such as the Beger (figure 1) or
- Frey procedure (figure 2) or
- pancreaticoduodenectomy (Whipple procedure),
- surgical drainage procedures, or
- endoscopic ductal decompression.

Options for surgical drainage can include procedures such as:


- the Puestow procedure (figure 3) or
- modifications of this classic operation such as the Izbicki operation or
- Hamburg or
- Berne procedure.
The Beger procedure includes
- duodenum-sparing resection of most of the pancreatic head
- with division of the pancreatic body over the portal vein and
- reconstruction via a side-to-side and side-to-end
pancreaticojejunostomy to drain the remaining head and tail of the
pancreas.

The Frey procedure combines a


- duodenal-sparing pancreatic head resection
- with overlay pancreaticojejunostomy and longitudinal duct drainage
without division of the pancreas.

The Puestow procedure consists of


- opening the duct in a longitudinal fashion and
- draining it via a lateral pancreaticojejunostomy
- while preserving the head of the gland.

Endoscopic treatment for pain related to chronic pancreatitis can be


accomplished with sphincterotomy and stone extraction with or without
ductal dilation or stent placement for main duct decompression.

A randomized controlled trial demonstrated that surgical drainage of the


pancreas
- required fewer reinterventions and
- was more effective in controlling pain and
- improving quality of life than endoscopic decompression.

Approximately 20–25% of patients required reintervention after endoscopic


decompression versus approximately 5% of surgically drained patients.

There were no statistically significant differences in reintervention rates


between patients undergoing Beger versus Frey procedures.

Rates of exocrine insufficiency are typically more than 60% in patients


undergoing either endoscopic decompression or surgery, and rates appear
similar between Beger and Frey procedures.

Rates of endocrine insufficiency appear to be lower than exocrine


insufficiency and are similar between endoscopic and surgical drainage
procedures.
Endocrine insufficiency is lower with pancreatic head parenchymal-sparing
surgical procedures than pancreaticoduodenectomy, but significant
differences have not been seen between the different types of pancreatic
parenchymal sparing operations.

Randomized controlled trials show superior results from duodenal-preserving


pancreatic head resections compared with pancreaticoduodenectomy.

Mortality from chronic pancreatitis may result from surgical complications if


operative drainage is performed; other causes include pancreatic cancer,
gastrointestinal hemorrhage, or cardiovascular disease as a result of
diabetes from endocrine insufficiency.

In fact, cardiovascular disease secondary to endocrine insufficiency is the


most common cause of death after Beger and Frey procedures.

Figure 1 Figure 2 Figure 3

Which of the following statements is true regarding necrotizing pancreatitis?


A
Contrast-enhanced CT is the imaging modality of choice to determine the
extent of necrosis.
B
Enteral nutrition should be avoided for 10–14 days.
C
Broad-spectrum antibiotics should be given.
D
Pancreatic necrosis mandates immediate surgical intervention.
E
Minimally invasive and percutaneous approaches to pancreatic necrosis are
contraindicated.

A
Contrast-enhanced CT is the imaging modality of choice to determine the
extent of necrosis.

Necrotizing pancreatitis is seen in approximately 1 in 5 patients with


pancreatitis and is associated with a mortality rate of 10–30%.

Patients are at high risk for multiorgan failure and secondary infection of the
necrotic pancreatic bed.

The risk of secondary infection is 25–70%, adding to the already high rate of
morbidity and mortality.

After initial admission and resuscitation, patients with severe pancreatitis


should undergo imaging to determine the extent of necrosis.

Contrast-enhanced axial imaging is the modality of choice.

The true extent of necrosis may not be apparent for several days.

CT offers the additional benefit of assessing for further local complications


associated with severe pancreatitis. 

Enteral nutrition should be started within 72 hours.

The goal is to achieve an early positive nitrogen balance.

If gastric feeds are not tolerated, continuous, jejunal feeds should be used.

In the setting of a paralytic ileus, parenteral nutrition can be used until


resolution of the ileus. 

Sterile necrosis must be distinguished from infected necrosis, because these


2 processes are managed differently.

Antibiotics have no apparent role in the setting of sterile necrosis.


Although some early studies supported the use of antimicrobial therapy in
the setting of severe necrosis, 2 large, randomized, double-blinded,
multicenter studies and a Cochrane meta-analysis determined prophylactic
antibiotics were not protective.

Infected necrosis warrants debridement.

There is no indication for surgical debridement of sterile necrosis.

Recently, several reports described the benefits of using a “step-up”


approach to debridement.

This approach advocates


- beginning with radiologic drainage,
- followed by minimally invasive retroperitoneal necrosectomy if
drainage fails.

This approach seeks to avoid the high morbidity and mortality associated
with open necrosectomy.

A 67-year-old man presents with a several-month history of vague


abdominal fullness and aching. He has episodes of flushing and
hypertension. He has been on tricyclic antidepressants for 5 years. He was
adopted as a child and does not know his relatives’ medical history.
Preliminary laboratory values are normal for complete blood count,
electrolytes, liver function tests, and clotting studies. A 24-hour urinary
catecholamine collection is elevated. An MRI is obtained and shown in figure
1. Which of the following statements is true?

FIGURE 1

A
This mass arises from the kidney.
B
Genetic testing is not recommended.
C
Obtaining plasma-free metanephrine off medication is the next appropriate
test.
D
Determination of the molecular marker p53 predicts the biologic behavior of
this mass.
E
Hypertension is associated with elevated cardiac index.

C
Obtaining plasma-free metanephrine off medication is the next appropriate
test.

This patient presents with an impressive mass in the retroperitoneal space


on the left.

Twenty-four-hour urinary catecholamine collection is elevated.

Therefore, unless proven otherwise, this patient has a pheochromocytoma.

The enlarged mass and the appearance of other nodules in the periaortic
area strongly suggest a malignant pheochromocytoma (figure 2).

This tumor does not arise from the kidney, and attention should be turned to
the appropriate preoperative workup and management of this patient in
preparation for surgery.

A pheochromocytoma is a tumor arising from the adrenomedullary


chromaffin cells that commonly produce one or more catecholamines,
including epinephrine, norepinephrine, and dopamine.

In this case, urine catecholamines are strongly suggestive of


pheochromocytoma, but this could be a false-positive.

This patient has been on tricyclic antidepressants for 5 years.

Therefore, checking peak values off the medication is clearly the best
answer.
Other medications that may cause falsely elevated tests for plasma and
urine metanephrines include
- acetaminophen,
- labetalol, and
- cocaine.

The literature states that 80–90% of patients with pheochromocytoma have


sustained or paroxysmal hypertension.

This is mostly due to increased peripheral resistance.

The hypertensive pattern in patients with pheochromocytoma is very similar


to those with primary hypertension with the exception of the tachycardia
these patients can experience.

Hypertension in pheochromocytoma is usually characterized by a high


peripheral resistance and low cardiac index.

Since 1990, 14 different susceptibility genes have been reported for patients
with pheochromocytoma.

Current guidelines recommend all patients with pheochromocytoma be


engaged in “shared decision making for genetic testing.”

This patient does not know his family history, and therefore, could possibly
harbor a genetic predisposition.
Identifying patients who may harbor the gene is of extreme importance for
offspring.
The p53 gene appears to have no role in predicting the behavior of
pheochromocytoma.
A 75-year-old woman with familial adenomatous polyposis who has had a
total colectomy complains of the new onset of intermittent abdominal pain,
which is worse with eating. An MRI shows 2 masses in the mesentery. Which
of the following is the most likely diagnosis?
A
Desmoid
B
Liposarcoma
C
Carcinoid
D
Lymphoma
E
Leiomyoma

A
Desmoid

The relative risk of developing desmoid tumors is much higher in patients


with familial adenomatous polyposis (FAP) compared with the general
population.

Desmoids occur in 7.5–16% of FAP patients.

Gardner syndrome, considered a variant of FAP, includes extracolonic


manifestations such as
- osteomas and desmoid tumors.
-
Given her history of FAP and the location of the masses, a diagnosis of
desmoid is most likely.

Diagnosis of desmoids and other aggressive fibromatosis (AF) is made based


on clinical, radiological, and histological parameters.

AF/desmoids characteristically infiltrate deep tissue and muscles as opposed


to pushing the adjacent tissue, as is seen in most sarcomas.
MRI is an excellent tool for the identification and characterization of
mesenteric masses (figure 1and table 1).

On MRI, the lesions are infiltrative with an irregular or lobulated contour.

Homogeneous isointensity or mild hyperintensity on T1-weighted images and


heterogeneous high signal on T2-weighted images is seen.

For symptomatic patients, treatment should be based on


- the location of the tumor and
- potential morbidity of treatment.

Treatment options include resection, radiation, or systemic therapy.

Radiation can be preoperative, intraoperative, or postoperative.

Systemic therapies for desmoids include


- anti-inflammatory drugs,
- hormonal (tamoxifen) agents,
- biologic agents (the tyrosine kinase inhibitor imatinib), or
- chemotherapy (doxorubicin).

The decision making for treatment recommendations should be


multidisciplinary and should include the preferences and quality-of-life
considerations of the patient.

Carcinoid tumors originate in the intestine.

Frequently, the lymph nodes in the mesentery are seen on scans; however,
their appearance is different, causing a sclerosing rather than infiltrative
appearance.

Carcinoids show arterial hyperenhancement and tethering of bowel loops,


whereas desmoids tend to show delayed hyperenhancement.

Lymphoma, while the most common solid neoplasm of the mesentery, is


usually associated with
- bulky adenopathy
- with preservation of fat around the mesenteric vessels, something
characterized as the “sandwich sign.”

Specifically, the nodes are dorsal and ventral to the mesenteric vessels,
appearing similar to “sandwich buns” with the mesenteric vessels and
surrounding fat appearing as the “sandwich meat.”
A liposarcoma would show up as mostly fat.

A leiomyoma is not usually present in the mesentery.

These tumors are difficult to differentiate from GIST tumors, because both
are mesenchymal tumors generally located within the muscularis mucosa or
submucosa.

They usually cause obstructive symptoms or may lead to ulceration and


bleeding.

Leiomyomas are not associated with FAP.

Regarding idiopathic thrombocytopenic purpura, which of the following


statements is true?
A
The spleen sequesters platelets.
B
Significant splenomegaly is usually present.
C
Long-term response to steroids is more than 50% in adults.
D
Diagnosis can be made using a peripheral blood smear.
E
The disease is usually self-limited for children younger than 5 years.

E
The disease is usually self-limited for children younger than 5 years.

Idiopathic thrombocytopenic purpura (ITP)


is a disease in which antibodies are formed against platelets, which results in
consumption of the platelets and thrombocytopenia.

These antibodies are created in the spleen, and a splenectomy is curative in


approximately 75–85% of patients.
ITP is associated with mucosal bleeding, petechiae, purpura, and
ecchymoses.

The diagnosis is made by the presence of antiplatelet antibodies seen on


serum testing.

For ITP, first-line medical therapy is the use of steroids to prevent the
formation of antibodies; 50–75% of adults will respond to the initial use of
steroid therapy, but long-term response rates are much lower at 15–20%.

If steroids do not improve the symptoms, a recurrence occurs, or serious


bleeding occurs, then anti-IgG antibody can be used.

In pediatric patients, ITP is self-limited in the majority of patients, especially


by the age of 5.
Transfusion, steroids, and anti-IgG antibody therapy is usually avoided
in pediatric patients unless the patient has life-threatening or
symptomatic bleeding.
When the platelet count is less than 20,000/mm3 (14,000–44,000/mm3),
pediatric patients are at risk for intracranial hemorrhage.

By contrast, thrombotic thrombocytopenic purpura (TTP) causes


thrombocytopenia, because the spleen sequesters the platelets and results
in splenomegaly.

TTP is associated with a


- low platelet count,
- hemolytic anemia,
- neurologic complications,
- renal failure,
- mental status changes, and
- bilateral lower extremity petechiae.

Here the diagnosis can be made by blood smear testing, which will show
- schistocytes,
- nucleated red blood cells, and
- basophilic stippling.

The treatment is usually plasmapheresis and transfusion of fresh frozen


plasma (not platelets), but splenectomy can be considered when the platelet
count and symptoms are refractory to plasmapheresis.

Splenectomy is curative in only approximately 40% of patients.


The most consistent predictor of response to splenectomy for idiopathic
thrombocytopenic (ITP) purpura is
A
response to steroids.
B
younger age.
C
larger spleen size.
D
degree of thrombocytopenia.
E
duration of ITP.

B
younger age.

Idiopathic thrombocytopenic purpura (ITP) occurs when autoantibodies


against platelet glycoprotein complexes result in varying degrees of
thrombocytopenia.

The spleen is the predominant site of antibody-induced platelet


sequestration and destruction.

Splenectomy is advocated in adults with thrombocytopenia from ITP with


reported long-term remission rates of 66–85% compared with much lower
remission rates reported from medical management (<30%).

The benefits of laparoscopic splenectomy reinvigorated the interest in


splenectomy as a treatment for ITP.

Many studies examined factors that predict a sustained improvement in


platelet counts after splenectomy.
Although the degree of response to steroids and shorter duration of disease
are positive predictors of response, the most consistent, independent
predictor is younger age.
There is no specific age cut off; the younger the patient is, the more likely
the chance of complete remission.

Lower platelet counts are a negative predictor.

A few studies show that spleen size does not predict response.

In general, patients with ITP have normal to slightly enlarged spleens.


Splenomegaly is uncommon.

Which of the following statements is true regarding the management of


sigmoid diverticulitis?
A
Colonoscopy is not needed after a first episode of uncomplicated
diverticulitis.
B
Routine elective resection in patients younger than 50 years is
recommended.
C
Colectomy followed by primary anastomosis with ileostomy may be the
optimal strategy for selected patients with perforated diverticulitis.
D
Routine elective resection should be performed in patients with 2 or more
episodes of uncomplicated diverticulitis.
E
Laparoscopic lavage is indicated for feculent peritonitis from perforated
diverticulitis.

C
Colectomy followed by primary anastomosis with ileostomy may be the
optimal strategy for selected patients with perforated diverticulitis.

Sigmoid diverticulitis is common, accounting for nearly 300,000 US hospital


admissions annually.
Evidence-based guidelines continue to evolve as our understanding of the
disease process improves.

Management has shifted from urgent operations for acute diverticulitis and
mandatory resection with colostomy formation to elective operations when
the surgeon is able to maintain intestinal continuity.

Prior guidelines supported routine elective resection in patients younger than


50 and in patients with 2 or more episodes of uncomplicated diverticulitis.

Newer evidence suggests that these groups are not at an increased risk of
complications with successive episodes and that the decision to proceed with
elective sigmoidectomy should be individualized.

An additional recommendation is that colonoscopy be performed within 6–8


weeks after an acute episode of diverticulitis if it is the first episode or if
colonoscopy was not performed recently.

Although this practice is debated, the concern is that a small subset of


patients (1–2%) will in fact have cancer, ischemia, or inflammatory bowel
disease.

Laparoscopic lavage is gaining popularity as an alternative surgical approach


for patients with Hinchey type II and III disease by CT imaging (table 1).

The procedure involves laparoscopic examination of the phlegmon and


irrigation of the abdomen but no disturbance of the inflammatory process.

Drains may or may not be placed.

Patients receive antibiotics and are followed clinically.

Critics argue that the risk of continuing or recurrent infection is a concern


when the septic focus remains.

Current data support laparoscopic lavage in Hinchey type III purulent


peritonitis, but no supportive data exist for laparoscopic lavage in Hinchey
type IV feculent peritonitis.

Traditionally, all patients who had an urgent operation for perforated


diverticulitis underwent resection with a colostomy.
This dogma is now challenged because of the high risk of complications
associated with colostomy takedown and colostomy closure rates are much
lower than ileostomy closure rates.

Thus, colectomy with a primary anastomosis and proximal ileal diversion for
Hinchey types III or IV disease may be optimal in select individuals versus
subjecting all patients to a routine Hartmann procedure.

Which of the following perioperative antibiotic regimens is associated with


the lowest risk of surgical site infection after colorectal surgery?
A
Oral antibiotics alone
B
Oral antibiotics and intravenous fluoroquinolone and metronidazole
C
Oral antibiotics and intravenous cefazolin and metronidazole
D
Oral antibiotics and intravenous cefazolin
E
Intravenous cefotetan alone

C
Oral antibiotics and intravenous cefazolin and metronidazole

Surgical site infection (SSI) are a common hospital-acquired infection,


causes increased morbidity, and adds significant costs to care.
Colorectal procedures are associated with a relatively high SSI rate (5–30%)
and thus are a prime target for reducing SSI.

The Surgical Care Improvement Project is a joint effort between the Centers
for Disease Control and Prevention and the Centers for Medicare and
Medicaid Services to develop a program to reduce SSI. 

In a Cochrane database analysis of 680 trials involving more than 43,000


patients, the investigators concluded that combined oral and intravenous
antibiotic prophylaxis reduced SSI compared with intravenous antibiotics
alone or oral antibiotics alone. 

In a review of elective colorectal procedures using the Veterans Affairs


Surgical Quality Improvement Program, 12% of almost 6000 patients
developed SSI.
Oral plus intravenous antibiotics (n = 2426) had a lower SSI rate than
intravenous antibiotics alone (n = 3324; 6.3% vs 16.7%, p <.0001).

A first-generation cephalosporin plus metronidazole was associated with


significantly lower SSI rates (6%) compared with ampicillin/sulbactam,
second-generation cephalosporins, or fluoroquinolone plus anaerobic
coverage (each >11%).

Oral antibiotic use was associated with a lower SSI rate for every class of
antibiotics used.

The most reliable method to detect small liver metastasis (<1 cm) from
colorectal carcinoma is
A
transabdominal ultrasound.
B
multidetector CT scan.
C
fludeoxyglucose-PET scan.
D
PET-CT scan.
E
contrast-enhanced MRI.
E
contrast-enhanced MRI.

Colorectal cancer is the third most common cancer in the United States, and
liver metastases occur in up to 30% of patients.

Hepatic resection of colorectal metastases may improve survival in select


patients; however, early detection and accurate imaging is crucial to
identifying the correct patients for treatment.
 
Due to its availability, ease of use, and relatively low cost (compared with
MRI), multidetector CT scan (MDCT) is generally the test of choice for
screening and staging of liver metastases.

These metastases are best detected by dynamic CT scanning during the


portovenous phase. 

Contrast enhanced MRI is the most sensitive imaging modality to detect


small (<1 cm) liver lesions due to its superior soft tissue resolution and the
ability to use diffusion-weighted imaging.

The sensitivity of contrast-enhanced MRI to detect liver metastases is 90–


95% compared with 70–75% for MDCT.

However, the increased costs and limitations of MRI (e.g., metal implants,
claustrophobia) limit its use as a screening tool.

The main role of MRI is in detection and characterization of small lesions.

In this respect, MRI outperforms both FDG-PET scan PET-CT scan in the
detection of small liver metastases.

Transabdominal ultrasound fails to detect more than 50% of liver


metastases.

Contrast-enhanced ultrasound may improve performance; however, this


method is not yet available in the United States.
The lowest rate of pancreatic leak after distal pancreatectomy is achieved
with
A
preoperative pancreatic stent placement.
B
suture ligation of the duct.
C
fibrin glue sealant applied to the transected duct.
D
stapler transection of the pancreas.
E
postoperative somatostatin analogue.

B
suture ligation of the duct.

The most common postoperative complication of distal pancreatectomy is a


leak from the cut edge of the pancreas, occurring in 20–30% of patients.

To avoid this leak and subsequent development of a pancreatic fistula, many


closure techniques and adjuncts were investigated. 

Direct suture ligation is the most reliable method to reduce pancreatic leak
after pancreatectomy.

Although initially thought to be promising, stapler transection of the


pancreas (with or without stapler reinforcement) does not reduce leak rate.

Postoperative pancreatic stent placement might be useful treatment for a


known leak; however, prophylactic use of this technique does not reduce
leak rate.

Routine use of fibrin glue sealant, closed suction drains, and postoperative
somatostatin analogue do not reliably reduce leak rates after distal
pancreatectomy.
Which of the following statements regarding overwhelming postsplenectomy
infection (OPSI) is true?
A
Patients undergoing emergency splenectomy for injury have worse OPSI
outcomes than patients having splenectomy for a hematologic disorder.
B
Asplenic patients are at risk for OPSI that is fatal in up to 25% of cases.
C
Appropriate initial antibiotic coverage for an asplenic patient with a fever
includes empiric treatment with vancomycin and ceftriaxone.
D
Splenic implants after splenectomy ensure protection against OPSI.
E
OPSI is most commonly caused by a Gram-negative organism.

C
Appropriate initial antibiotic coverage for an asplenic patient with a fever
includes empiric treatment with vancomycin and ceftriaxone.

Asplenic or hyposplenic patients have increased risk for infection and death
from encapsulated organisms, often pneumococcus, Haemophilus
influenza type b, or meningococcus.

However, patients who have a splenectomy to treat an underlying


hematologic outcome are at greater risk for postsplenectomy infection.

Splenic implants are not considered adequate protection against


overwhelming postsplenectomy infection (OPSI).

OPSI can progress rapidly from a mild flu-like illness to fulminant sepsis that
is fatal in up to 70% of cases with delayed or inadequate treatment.

Because of the high mortality and fulminant course associated with OPSI,
vaccination and antibiotic prophylaxis are the basis of the management of
asplenic or hyposplenic patients (figure 1).
Pneumococcal, H. influenzae type b, meningococcal, and influenza virus
vaccinations are recommended for asplenic patients, preferably 2 weeks
before an elective surgery.

A booster dose should be administered every 5 years.

Children who receive their dose before age 7 should receive a booster dose 3
years later with subsequent doses every 5 years.

Asplenic patients with a fever should receive empirical antimicrobial therapy.

Rapid identification of patients at risk for OPSI and administration of


vancomycin and ceftriaxone to cover Streptococcus pneumonia, H. influenzae,
Neisseria meningitidis, and many community-acquired Gram-negative bacilli
can improve survival.

Prophylactic antimicrobial therapy is recommended for asplenic children


younger than age 5 and may be considered for older children and adults for
1–2 years after splenectomy.

Patients who have had postsplenectomy infection should have lifelong


prophylaxis.

Asplenic patients should be educated about the risk of life-threatening


infection with any illness with fever or severe flu-like symptoms without
fever.
A 38-year-old woman presents with right upper quadrant pain, postprandial
gas, and bloating associated with nausea. Gallbladder ultrasound shows a
distended gallbladder with no stones. Which of the following statements is
most accurate regarding cholecystokinin cholescintigraphy?
A
It is useful when patients describe recurrent epigastric or right upper
quadrant pain episodes lasting 30 minutes or longer.
B
It is useful in determining the etiology of atypical abdominal pain.
C
Symptom reproduction is the most useful predictor of right upper quadrant
symptom relief by cholecystectomy.
D
H2 receptor antagonists increase gallbladder contractility and may produce a
false-negative test.
E
An abnormal gallbladder ejection fraction is highly specific for gallbladder
disease.

A
It is useful when patients describe recurrent epigastric or right upper
quadrant pain episodes lasting 30 minutes or longer.

Cholecystokinin (CCK) cholescintigraphy is an appropriate step in diagnosing


the patient with typical gallbladder symptoms who has a negative ultrasound
evaluation.

CCK or a synthetic derivative (sincalide) is injected 30–60 minutes after the


administration of Tc-99m labeled disofenin (DISIDA, 2,6-diisopropyl
acetanilidoiminodiacetic acid) or mebrofenin (BRIDA, bromo-2, 4,6-trimethyl
acetanilidoiminodiacetic acid).

Imaging starts after the injection and continues for 60 minutes.

When acute cholecystitis is suspected and the gallbladder is not seen within
60 minutes, imaging should be continued for up to 3–4 hours.

Imaging for 18–24 hours may be necessary in some cases (e.g.,


- a severely ill patient,
- severe hepatocellular dysfunction, or
- suspected common bile duct obstruction).

If the patient is being studied for a biliary leak, 2- to 4-hour delayed imaging
should be obtained.
The Society of Nuclear Medicine defines an ejection fraction of less than 38%
as abnormal. This is a calculated number designated as 2 standard
deviations from the mean and there will be “normal” patients who have
ejection fractions less than 38% and abnormal patients who have ejection
fractions greater than 38%.

Thus, specificity of this test for gallbladder disease is low.

The use of CCK cholescintigraphy for diagnosis of biliary tract disease should
be limited to those patients who meet ROME III criteria (table 1) for
functional gallbladder disorders.

It should not be used to evaluate atypical abdominal pain, because many


medical conditions, including diabetes, irritable bowel disease, and celiac
disease, can produce an abnormal gallbladder ejection fraction.

Several medications may reduce gallbladder contractility and should be


withheld before the test.

These include
- atropine,
- calcium channel blockers,
- octreotide,
- progesterone,
- indomethacin,
- theophylline,
- benzodiazepines,
- H2antagonists, and
- all opioids.

Although many physicians rely on reproduction of the patient’s symptoms by


the CCK infusion to recommend cholecystectomy, this is a very nonspecific
finding.

Several small retrospective studies recommended removal of the gallbladder


for symptoms induced by CCK infusion, but at this time it is not considered a
reliable test for the effectiveness of cholecystectomy in curing symptoms.

Reproduction of symptoms is quite often the result of rapid infusion of the


CCK analog
=====================================================================

A 22-year-old woman develops acute cholecystitis during the 32nd week of


pregnancy. Which of the following statements regarding her management is
true?
A
Premature labor is less common than in the second trimester.
B
Intraoperative cholangiography is associated with an adverse fetal outcome.
C
Open cholecystectomy is safer than laparoscopic cholecystectomy.
D
Postoperative endoscopic retrograde cholangiopancreatography is safe.
E
Gallstone pancreatitis is associated with a fetal loss of 20%.
D
Postoperative endoscopic retrograde cholangiopancreatography is safe.

The most appropriate time to (semi-) electively operate on a pregnant


women with cholecystitis is in the second trimester (13–26 weeks), because
there is a lower incidence of premature labor compared with the third
trimester.

At 32 weeks, conservative management of her cholecystitis should be


attempted initially.

If an operation is mandated, the mother should be pretreated with


betamethasone 24 hours and 12 hours preoperatively.

This approach enhances surfactant production and reduces the need for
neonatal respiratory support. 

At 32 weeks, any radiographs that are clinically needed are safe.


This includes
- intraoperative cholangiography as well as
- endoscopic retrograde cholangiopancreatography (ERCP).
- The fetus should be shielded when possible.
- External fetal monitoring is indicated. 

Laparoscopic surgery is as safe or safer than an open procedure in


pregnancy.

An open cutdown to the peritoneal cavity is indicated (Hasson technique).

Gallstone pancreatitis is associated with a fetal loss of 4.7% compared with


a fetal loss of 2.8% with non–gallstone pancreatitis.

A patient with gallstone pancreatitis should be treated with a


cholecystectomy and clearing of stones with operative common duct
exploration or postoperative ERCP.

Both are associated with lower rates of fetal loss than conservative
management.

A 48-year-old woman with known cirrhosis from prior alcohol abuse develops
increasing confusion. Her blood ammonia levels are elevated. A diagnosis of
hepatic encephalopathy is made secondary to hepatic failure. The best
definitive treatment is
A
portacaval shunt.
B
protein restriction.
C
splenorenal shunt.
D
transjugular intrahepatic portosystemic shunt.
E
liver transplantation.

E
liver transplantation.

The only definitive treatment for end-stage hepatic failure is liver


transplantation.

Portacaval shunting, splenorenal shunting, and a transjugular intrahepatic


portosystemic shunt (TIPS) will all reduce portal hypertension, which will
reduce gastrointestinal bleeding, but they do not treat the underlying liver
failure and will exacerbate encephalopathy.

Although protein restriction was previously recommended for hepatic


encephalopathy, it is no longer indicated and may be harmful.

High-protein diets are well tolerated in patients with cirrhosis; they should
receive 1–1.5 g/kg of protein and 25–40 kCal/kg per day.

Most patients with cirrhosis have a deficiency of branched chain amino acids.

These are commonly found in dairy products and vegetables.

Augmentation of the diet with these proteins may be useful.


Although there is anecdotal evidence in favor of the use of purgative agents
such as nonabsorbable disaccharides (such as lactulose), no comprehensive
meta-analysis has shown them to be effective.

A 34-year-old man is diagnosed with a transverse colon cancer in the setting


of multiple colon polyps. There is no prior family history of colon cancer. He
has a total proctocolectomy with an ileal pouch anal anastomosis. His tumor
is a T3N0M0, and more than 100 other polyps were noted throughout his
colon and rectum. Two years later, he has a surveillance CT scan, and a 3-
cm mesenteric mass is noted. A core needle biopsy shows very bland-
appearing spindle cells. Which of the following is the next step in
management of this patient? 
A
Sulindac
B
Imatinib
C
FOLFOX (5-fluorouracil, leucovorin, and oxaliplatin)
D
Radiation
E
Surgical excision

A
Sulindac

This patient has clinical evidence of familial adenomatous polyposis (FAP).

Patients with classic FAP are at high risk of developing abdominal desmoid
tumors, occurring in approximately 10–15% of patients with FAP.

Attenuated FAP is a subtype and recognized by fewer polyps, presentation at


an older age and less risk for developing desmoid tumors. 

Desmoid tumors are considered benign, because they do not metastasize.

However, despite having a benign designation, abdominal desmoid tumors


can cause significant local problems as they grow into surrounding tissues.
Therefore, while considered “benign,” some abdominal desmoid tumors
clearly have a “malignant” effect on patient’s lives and can result in death.

The natural history of these tumors is unpredictable, with a small number


regressing completely, some waxing and waning, many staying stable over
time, and a small number rapidly progressing.

The management of these lesions is multidisciplinary.

Surgery is reserved for tumors that are symptomatic and resectable,


because recurrence is common.

Simple observation is reasonable for small, asymptomatic lesions.

As part of this strategy, serial CT scans to identify complications, such as


ureteral obstruction, is reasonable. 

First-line pharmacologic agents include


- nonsteroidal anti-inflammatory medications (e.g., sulindac) and
- antiestrogens (e.g., tamoxifen).

Cytotoxic agents can be used for advanced disease that is not responding to
these other, less-toxic choices.

Surgery is often complicated and associated with frequent complications.

Therefore, it is done only for significant symptoms.

When necessary, resection is the best option if possible.

However, many patients will have unresectable disease, so a bypass can be


done to relieve a bowel obstruction. 

This patient has an incidental finding of a mesenteric mass.

While at risk for metastatic colon cancer, the biopsy is most consistent with
a desmoid tumor.

Resection is not indicated at this time and should not be done.

Imatinib is a tyrosine kinase inhibitor used to treat gastrointestinal stromal


tumors.
FOLFOX (5-fluorouracil, leucovorin, and oxaliplatin) is a combination
chemotherapeutic regimen used for the adjuvant treatment of colon cancer.
Radiation is not used in the treatment of desmoid tumors, because it
appears ineffective and is associated with a high rate of complications.
A 47-year-old woman presents with acute onset of abdominal pain. CT
shows evidence of acute portal vein thrombosis (PVT). Which of the following
is true regarding acute PVT? 
A
In most patients, only the portal vein is involved.
B
A myeloproliferative disorder is rarely associated with PVT.
C
Clinically significant ascites is uncommon.
D
Recanalization occurs frequently without anticoagulation.
E
Common initial presentation is gastrointestinal bleeding from varices.

C
Clinically significant ascites is uncommon.

Acute portal vein thrombosis often presents with onset of vague abdominal
pain.

Imaging studies such as CT will typically define the significance of the


abnormality.

In most patients, the portal vein and its tributaries and branches are
involved.

One study found that more than 50% of the patients had a prothrombotic
state, and 35% had a myeloproliferative disorder as a precipitating factor in
the development of portal vein thrombosis. 

Imaging often shows ascites, but it is rarely clinically significant.


Gastroesophageal varices develop in up to 50% of patients without
recanalization, although this is rarely an initial presentation.

Studies suggest that gastroesophageal varices may develop as early as a


month after thrombosis but more commonly develop later over time.
Early anticoagulation is associated with recanalization in 40% of patients
studied, but no patients recanalized when anticoagulation was not used.

Another study found similar findings in patients who underwent early


anticoagulation that continued for a mean of 8 months; 39% patients
developed recanalization.

Early anticoagulation of patients with acute portal vein thrombosis leads to a


better outcome with a reduced risk of gastrointestinal complications.

A 57-year-old woman presents for acute symptoms of right upper quadrant


pain. The CT scan obtained is shown in figure 1 and figure 2. Which is the
following is the most appropriate treatment?

FIGURE 1

FIGURE 2

A
Estrogen therapy
B
Somatostatin
C
Sclerotherapy
D
Fenestration
E
Hepatic resection

D
Fenestration

This patient presents with acute symptoms related to her polycystic liver
disease.

Indications for intervention include


- acute pain from potential bleeding into the cyst,
- vague mass effect from the size of the cyst, and,
- rarely, liver dysfunction.

Fenestration is most effective, can be done laparoscopically, and often is


performed as an outpatient procedure.

There is minimal risk of bleeding or postoperative complications with this


approach. 

Hepatic resection is not necessary in the management of this disease.

Sclerotherapy is ineffective with a high rate of recurrence.

Observational studies suggest that pregnancy and estrogen replacement


therapy can increase the number and volume of cysts.

Somatostatin analogues may reduce the overall volume of the liver but have
no effect on the size of the cysts.

A 35-year-old man develops acute necrotizing pancreatitis related to a drug


reaction. The CT scan shown in figure 1 and figure 2 was obtained 1 month
after the acute process. Although he is afebrile, he is having trouble eating,
has decreased stamina, and has moderate discomfort to palpation of the
right lower quadrant. His white blood cell count is mildly elevated. Which of
the following is the most appropriate management in this patient?

FIGURE 1

FIGURE 2

A
Expectant management for continued resolution
B
Cyst gastrostomy
C
Percutaneous aspiration
D
Transabdominal debridement
E
Retroperitoneal debridement

E
Retroperitoneal debridement

The management of acute necrotizing pancreatitis continues to evolve.


A multitude of different modalities are now available that affect the timing
and management approach for patients with walled-off pancreatic necrosis
(WOPN).

This patient continues to be symptomatic a month after his acute process.

The CT scan suggests a small fleck of air in one of the pockets of the WOPN
(figure 3).

Expectant management of this patient may continue to work, although his


recovery is likely to be slow.

The gas within the fluid collection raises concern for bacterial contamination
and infection.

There is not an obvious endoscopic site for transluminal drainage of this fluid
collection, and the multiple loculated areas would make this less likely to
work effectively.

Percutaneous aspiration is a diagnostic test to exclude an infected fluid


collection.

Percutaneous drainage is possible and may expedite resolution of the


patient’s symptoms, but often the material cannot be completely evacuated.

Laparoscopic debridement runs the risk of further seeding infected fluid


throughout the peritoneal cavity, which would be less than ideal.

Direct percutaneous endoscopic debridement, also referred to as direct


retroperitoneal debridement, offers an excellent option for management of
this process.

The procedure uses laparoscopic instruments passed in a retroperitoneal


plane into the cavity for debridement and irrigation or direct percutaneous
endoscopic debridement.

This procedure is usually facilitated by first passing image-guided catheters


into the cavity and using these catheter tracks to guide the laparoscopic
instruments.
Endoscopic debridement was used in this patient with prompt resolution of
the symptoms and no need for subsequent operative intervention.

Which of the following is a component of both the Child-Pugh and Model for
End-Stage Liver Disease scoring systems?
A
Albumin
B
Aspartate transaminase
C
Creatinine
D
Encephalopathy
E
International normalized ratio

E
International normalized ratio

The Child-Turcotte-Pugh (CTP) score was originally developed to evaluate


the risk of portocaval shunting procedures in patients with portal
hypertension.

It is also used to evaluate surgical risk in other intra-abdominal procedures


performed in cirrhotic patients.

Overall surgical mortality is


- 10% for patients with class A cirrhosis,
- 30% for those with class B cirrhosis, and
- 75–80% for those with class C cirrhosis.
Components of the CTP score are
- international normalized ratio (INR),
- bilirubin level,
- albumin level,
- encephalopathy,
- ascites (table 1).

The CTP score is criticized for its 2 subjective variables (degree of


encephalopathy and ascites), narrow range (5–15), and the equal weight
given to each variable.

The Model for End-Stage Liver Disease (MELD) is a linear regression model
based on 3 laboratory values
o INR,
o bilirubin, and
o creatinine level

It was initially developed as a model to predict mortality after transjugular


intrahepatic portosystemic shunt but has now become the method used for
liver transplant allocation in the United States.

The MELD formula is shown in figure 1.


MELD is an excellent predictor of postoperative mortality.

One study demonstrated an


- increase in mortality of 1% for each MELD point, up to a score of 20,
and by
- 2% for each MELD point above 20.

Cirrhotic patients undergoing urgent surgery or major operations have a


relative risk of mortality increase of 14% for each 1-point score in their
MELD score (figure 2).

Both the CTP and MELD scoring systems use INR and total bilirubin in their
calculation.
=====================================================================

An 82-year-old man presents with a 7-day history of right upper quadrant


pain with nausea and vomiting. His medical history is remarkable for
hepatitis C and cirrhosis. His white blood cell count is 18,500/mm 3 (3600–
11,200/mm3) with 13% bands. Total bilirubin is 5.5 mg/dL (0.2–1.9 mg/dL),
and alkaline phosphatase is 325 units/L (<95 IU/L). CT scan of the abdomen
and pelvis demonstrates severe cholecystitis with pericholecystic fluid,
cholelithiasis, and a normal common bile duct. The most appropriate next
step should be
A
magnetic resonance cholangiopancreatography.
B
hepatobiliary iminodiacetic acid scan.
C
endoscopic retrograde cholangiopancreatography.
D
percutaneous cholecystostomy.
E
open cholecystectomy.

D
percutaneous cholecystostomy.

Acute calculous cholecystitis


is a common surgical emergency,
often affecting elderly patients and those with serious medical comorbidities.
Emergency cholecystectomy in this high-risk group of patients is associated
with increased morbidity up to 41% and with mortality up to 4.5%.

An alternative treatment is image-guided percutaneous cholecystostomy


under local anesthetic, which has less morbidity and mortality than open or
even laparoscopic cholecystectomy in the emergency setting.

Once the acute episode is resolved, cholecystectomy may or may not be


considered as an elective intervention.

Magnetic resonance cholangiopancreatography (MRCP) requires no radiation


and is a noninvasive way to visualize the biliary tree.

This study would not be done before cholecystostomy, but may be indicated
to exclude choledocholithiasis later. 

Hepatobiliary iminodiacetic acid (HIDA) scan is a nuclear medicine study that


uses the nuclear colloid, iminodiacetic acid.
The nuclear colloid is injected intravenously, processed in the liver, and
secreted within the bile.

Acute cholecystitis is confirmed if there is no visualization of the gallbladder


or failure to fill the gallbladder within 2 hours after injection, which is
considered evidence that the cystic duct is obstructed.

HIDA scan is not needed in this patient to make the diagnosis of acute
cholecystitis given the CT findings.

Endoscopic retrograde cholangiopancreatography (ERCP) is an invasive


procedure in which contrast is injected through the ampulla to image the
biliary tree.

ERCP can both diagnose and treat biliary obstruction secondary to


choledocholithiasis, but it will not treat acute cholecystitis.

Which of the following statements regarding colorectal liver metastases is


true? 
A
Colorectal metastases represent the third most common malignant tumor of
the liver.
B
Resectability is based on the volume of liver remaining after resection.
C
Four or more hepatic metastases are a contraindication to liver resection.
D
Radiofrequency ablation is appropriate for metastatic tumors larger than 5
cm.
E
Neoadjuvant chemotherapy yields best results when used for single
colorectal metastases.

B
Resectability is based on the volume of liver remaining after resection.

Liver metastases from colorectal cancer are the most frequent hepatic
malignancies in the United States.

Radiofrequency ablation (RFA) is a local therapy that uses a form of


alternating electrical current to achieve thermal destruction.
RFA is performed using several techniques: percutaneous, open, or
laparoscopic.

Tumor size and location can preclude effective RFA when used as curative
treatment.

Tumor sizes larger than 4–5 cm are associated with an increased incidence
of recurrence. 

Traditionally, the presence of 4 or more hepatic metastases was a


contraindication to hepatectomy.

Current data reveal that hepatectomy for 4 or more metastases is associated


with an approximate 5-year survival rate of 33%.

Resectability is based on volume of liver remaining after resection and not


the actual number of tumors.
- In a patient with normal liver function, a 20% remnant is
recommended.
- In a patient who has undergone neoadjuvant chemotherapy, a 33%
remnant is recommended. 
Neoadjuvant chemotherapy can yield good results when used for multiple
colorectal metastases.

Chemotherapy and complete resection of hepatic metastases are associated


with long-term survival in up to 50–60% of patients.

An 80-year-old woman is admitted to the medical intensive care unit with


pneumonia and sepsis. She is intubated and on norepinephrine and
vasopressin for 5 days without improvement. Current therapy includes
piperacillin/tazobactam and vancomycin. Her liver function tests are
elevated, with a total bilirubin of 1.8 mg/dL (0.2–1.9 mg/dL), aspartate
aminotransferase of 60 U/L (women 8–40 IU/L), alanine aminotransferase of
50 U/L (7–55 IU/L), and alkaline phosphatase of 100 U/L (<95 IU/L). An
ultrasound of her right upper quadrant demonstrates an enlarged gallbladder
with pericholecystic fluid and wall thickening, but no gallstones. The common
bile duct measures 4 mm. Which of the following is the most appropriate
next step in the management of this patient?
A
Open cholecystectomy
B
Percutaneous cholecystostomy tube placement
C
Endoscopic retrograde cholangiopancreatography
D
Observation
E
Magnetic resonance cholangiopancreatography

B
Percutaneous cholecystostomy tube placement

This patient has acalculous cholecystitis, and no further imaging is necessary


to confirm the diagnosis.

Acalculous cholecystitis is typically associated with critical illness, and most


of these patients are poor surgical candidates.
Mortality associated with this disease is 41%.

Acalculous cholecystitis is generally the result of biliary stasis and


gallbladder ischemia, although the pathophysiology is incompletely
understood.

Stasis is frequently due to prolonged fasting or hyperalimentation.

Progression of disease with gallbladder complications, such as perforation,


gangrenous cholecystitis, or emphysematous cholecystitis, occurs in 40–
100% of these patients; therefore, observation alone is not recommended.

Magnetic resonance cholangiopancreatography or endoscopic retrograde


cholangiopancreatography would not benefit this patient, because the
common bile duct is normal in caliber and gallbladder inflammation and
ischemia is the source of her sepsis.

Percutaneous cholecystostomy is recommended as definitive therapy or as a


bridge to ultimate cholecystectomy when the patient is medically stable.

In a hemodynamically normal patient, most authors recommend a


laparoscopic approach initially rather than an open approach given the
proven safety and decreased hospital length of stay of the former.

Which of the following statements is true regarding nonparasitic splenic


cysts?
A
Management should consist of splenectomy for cysts greater than 5 cm in
diameter.
B
Percutaneous needle aspiration is often definitive therapy.
C
Symptom relief after percutaneous needle aspiration may predict response
to operative management.
D
Most nonparasitic splenic cysts are symptomatic.
E
There is a high incidence of spontaneous or posttraumatic rupture of splenic
cysts.

C
Symptom relief after percutaneous needle aspiration may predict response
to operative management.

In the United States, nonparasitic splenic cysts may be posttraumatic or


primary (figure 1).

Rupture of splenic cysts either spontaneously or posttraumatically is rare.

Studies suggest that fewer than 60% of posttraumatic cysts are


symptomatic.

Few data support the use of cyst size as an indication for splenectomies in
asymptomatic patients.

Postaspiration cyst and symptom recurrence are very common; however,


symptomatic relief with aspiration can suggest a benefit of operative
management of splenic cysts.

Because of the benign nature of the majority of splenic cysts, partial


splenectomies or partial cystectomies (unroofing) are reported in most
patients.

Which of the following statements is true regarding the management of


patients with achalasia?
A
Overall symptom relief is higher with Heller myotomy and partial gastric
fundoplication than with per-oral endoscopic myotomy (POEM).
B
The incidence of technical complications after POEM exceeds that seen after
laparoscopic Heller myotomy and partial gastric fundoplication.
C
POEM and Heller myotomy achieve complete ablation of the lower
esophageal sphincter complex.
D
POEM can achieve similar rates of postprocedural symptomatic esophageal
reflux to those obtained with laparoscopic Heller myotomy and partial gastric
fundoplication.
E
Laparoscopic Heller myotomy is combined with a partial gastroesophageal
fundoplication to eliminate postoperative gastroesophageal leaks.

D
POEM can achieve similar rates of postprocedural symptomatic esophageal
reflux to those obtained with laparoscopic Heller myotomy and partial gastric
fundoplication.

Per-oral endoscopic myotomy (POEM) divides only the circular muscle fibers
of the lower esophagus and stomach.

It is proposed that this incomplete ablation of the lower esophageal complex


compared with the Heller procedure lowers the 20–100% incidence of
gastroesophageal reflux that accompanies the Heller procedure and
necessitates a gastroesophageal fundoplication in combination with a full-
thickness myotomy.

A 2014 study compared consecutive groups of patients treated with the 2


procedures.

Technical complication rates were similar with the 2 procedures.

Successful treatment of dysphagia with POEM, and rarely symptomatic


gastroesophageal reflux, was comparable to the results with Heller myotomy
and partial gastric fundoplication.
The purpose of adding partial gastroesophageal fundoplication to a Heller
myotomy (circular and longitudinal muscle layers) is to prevent
gastroesophageal reflux after this procedure (video 1) PEOM

Which of the following statements is true about carcinoid tumors of the


appendix?
A
The majority present with carcinoid syndrome.
B
The depth of tumor invasion is the most accurate predictor of lymph node
metastasis.
C
The status of regional lymph nodes is the single most important factor
influencing 10-year survival.
D
The tumor size is the most accurate predictor of lymph node metastasis.
E
Appendectomy alone is adequate treatment for tumors less than 4 cm in
diameter.

D
The tumor size is the most accurate predictor of lymph node metastasis.

The majority of appendiceal carcinoid tumors are small, and they are
discovered incidentally.

Lymph node metastases from the tumors are predicted most accurately by
size, with tumors less than 1 cm in diameter and confined to the appendix
not associated with lymph node metastases and treated with appendectomy
alone.

Tumors greater than 2 cm in diameter are treated with right hemicolectomy.

Controversy surrounds the management of patients with tumors 1–2 cm in


diameter as well as carcinoids at the base of the appendix.
The status of regional lymph nodes does not influence 10-year survival.

Because the majority of tumors are less than 1 cm in diameter when they
are discovered and unlikely to be associated with metastatic disease, the
incidence of carcinoid syndrome is very rare in patients with appendiceal
carcinoid tumors.

A 44-year-old man with poorly controlled diabetes was diagnosed with


severe pancreatitis and associated pancreatic necrosis (figure 1). He was
managed without intervention and discharged after 6 days. He re-presents 2
weeks later with increasing diffuse abdominal pain. He is normotensive with
a heart rate of 105 beats per minute and a temperature of 38.3°C. He has
diffuse abdominal tenderness without peritoneal signs. His white blood cell
count is 24,000/mm3 (3600–11,200/mm3), his hemoglobin is 10 g/dL (men
13.5–17.5 g/dL), and his serum glucose is 380 mg/dL (70–100 mg/dL).
Repeat imaging is seen in figure 2. Which of the following is the next most
appropriate step in this patient’s management?

FIGURE 1 FIGURE 2

A
Admission with nasogastric tube placement, bowel rest, and octreotide
B
Retroperitoneal debridement
C
Percutaneous drain placement
D
Exploratory laparotomy
E
Endoscopic placement of distal feeding access for enteral nutrition
C
Percutaneous drain placement

This patient has evidence of infected pancreatic necrosis (figure 3).


Up to 30% of patients develop infection after necrotizing pancreatitis, with
presentation on average 3–4 weeks after the original management of
pancreatitis.
In this case, the extraluminal gas evident on CT imaging is suggestive of the
diagnosis, even without culture results.

Management of infectious complications after pancreatitis and pancreatic


necrosis has evolved in recent years from mandated laparotomy and open
necrosectomy.

Recent reports demonstrate that percutaneous drainage may control sepsis


and avoid surgery in more than 50% of patients; if the patient has ongoing
sepsis uncontrolled by percutaneous drainage, surgical intervention should
be entertained.

In those cases, the percutaneous drain can serve as a “road map” to direct a
minimally invasive approach, such as the step-up approach or video-assisted
retroperitoneal debridement.

Minimally invasive surgical approaches have


- lower morbidity (e.g., multiple organ failure, incisional hernia, new
onset diabetes) and
- mortality rates
compared with open necrosectomy.

Enteral nutrition tends to improve outcomes for pancreatitis, so indefinite


bowel rest is not advocated.

Antibiotics alone for a fluid collection of this size, particularly in a patient


with sepsis, is not advocated.
Distal feeding access may be desirable, particularly for those patients with a
gastric ileus, but that would not be the first step in management.

=====================================================================

A 60-year-old man presents with epigastric pain radiating to his back and
nausea. On exam, he has focal epigastric tenderness. He has elevated
transaminases, a lipase of 1800 units/L (reference is 10–40 units/L), and
normal bilirubin. Right upper quadrant ultrasound demonstrates
cholelithiasis and a common bile duct measuring 9 mm. After admission to
the hospital, intravenous fluid resuscitation, and pain medication, his pain
resolves and laboratory values normalize. Which of the following is the next
step? 
A
Laparoscopic cholecystectomy
B
Endoscopic retrograde cholangiopancreatogram
C
Discharge home with outpatient follow-up
D
Magnetic resonance cholangiopancreatogram
E
Laparoscopic cholecystectomy at 6 weeks

A
Laparoscopic cholecystectomy

Gallstones and alcohol are the most common causes of acute pancreatitis.

The presence of high serum lipase and typical abdominal pain suggests the
diagnosis in the absence of a CT scan.
The presence of gallstones by abdominal ultrasound suggests the etiology,
which can be further supported by the presence of a dilated common bile
duct and the finding of elevated serum transaminases.

The vast majority of stones pass spontaneously within 48 hours of


presentation.

Mild cases can be treated with hospital admission, fluid resuscitation, and
pain control.
In patients with more severe disease,
- cholangitis,
- persistent hyperbilirubinemia,
- clinical deterioration, or
- detection of a persistently impacted common bile duct stone,
endoscopic retrograde cholangiopancreatogram (ERCP) is warranted within
24–48 hours.

In patients with resolving symptoms of gallstone pancreatitis,


cholecystectomy is indicated to remove the source of stones and prevent
recurrence.

Delayed cholecystectomy is associated with a high rate of recurrence of


pancreatitis.
A randomized trial demonstrated that early laparoscopic
cholecystectomy is associated with
- a shorter overall length of hospital stay
- without increasing operative complications
- or conversion rates, compared with delayed cholecystectomy.

Magnetic resonance cholangiopancreatogram (MRCP) does not offer


therapeutic options and is reserved for cases that have an equivocal
diagnosis or anatomy.

A 60-year-old man with known chronic liver disease secondary to alcohol


abuse presents with acute onset right upper quadrant abdominal pain. He is
diaphoretic. His blood pressure is 90/40 mm Hg, his heart rate 120 beats per
minute, and his respiratory rate 20. Resuscitation is initiated. After
improvement of his vital signs, he undergoes a CT scan that shows
hemorrhage from a 4-cm liver mass (figure 1). Therapy should begin with
which of the following?
FIGURE 1

A
Biopsy of the mass
B
Transjugular intrahepatic portosystemic shunt
C
Embolization
D
Laparotomy and packing
E
Mass resection

C
Embolization

This individual is in a high-risk group for hepatocellular carcinoma (HCC),


and this presentation is likely a ruptured HCC.

A biopsy is unnecessary and unsafe at this point.

The immediate goal is control of the bleeding.

The diagnosis may be confirmed based on the result of the recommended


therapeutic arteriography.

The patient’s abnormal hemodynamics and active bleeding warrant urgent


intervention.
HCCs have a rich arterial supply, and embolization is the preferred first
intervention in the setting of acute rupture.

The expected success rate would be better than 80%.

The bleeding is clearly arterial, as evidenced by the blush seen on CT scan


(figure 2), so transjugular intrahepatic portosystemic shunt (TIPS) would not
be indicated.

TIPS would be considered in the setting of bleeding from esophageal varices


refractory to endoscopic intervention.

Surgical approaches to this presentation—resection or packing—carry a


significant risk of morbidity and mortality in the acute setting in patients
with chronic liver disease and should be reserved for patients failing
embolization.

A 55-year-old man had an esophagectomy for adenocarcinoma of the


esophagus (stage IIa). Six months later, he is found to have 3 technically
resectable liver metastases 3–6 cm in diameter. Which of the following
statements is true about liver-directed therapy for this patient’s
presentation?
A
Resection confers a survival advantage over chemotherapy.
B
Radiofrequency ablation has a low local failure rate.
C
External beam radiation is curative.
D
Systemic chemotherapy should be offered.
E
Transarterial chemoembolization is the preferred care for hepatic metastases
of noncolorectal, nonneuroendocrine cancers.

D
Systemic chemotherapy should be offered.

In this patient, there are several contraindications to liver-directed therapy.


- The short disease-free interval and
- presentation with multiple lesions suggests poor tumor biology, or
- inadequate initial staging, or both.

Metastatic esophageal adenocarcinoma is rarely confined to the liver;


therefore, systemic therapy is indicated.

There is no evidence that resection has added benefit over systemic


chemotherapy.

Noncolorectal, nonneuroendocrine cancers metastatic to the liver are a


heterogeneous group of diagnoses with highly variable biology.

Few tumor types, other than colorectal carcinoma, are known to metastasize
to the liver in a manner amenable to curative intent intervention.

Most studies show no benefit of resection or ablation of liver metastases in


the treatment of gastrointestinal malignancies. 

The interest in expanding the indications for liver-directed therapies has


increased with the advent of new technologies and the improved safety of
liver resection.

Tumor biology is still the principle harbinger of successful treatment in any


given patient.

The treating physicians should have a clear understanding of the goals of


treatment (curative vs. palliative intention) as well as the patient’s
preferences.

The ability to perform a procedure alone is insufficient to serve as an


indication.
Radiofrequency ablation and other percutaneous thermal ablation techniques
enjoyed popularity in the past decade due to ease of delivery and generally
favorable safety profiles.

The limitations of the technology include tumor number and size.

The ability to reliably and safely destroy tumors larger than 3 cm is in


question, with local failure rates in tumors larger than 5 cm being greater
than 30%. 

External beam radiation is an attractive option given recent advancements in


tumor localization and delivery.

The literature supports a role in palliation of symptomatic lesions but not in


curative intent treatment.

Likewise, transarterial chemoembolization has a theoretical role in palliation


of vascular lesions but is not a standard for any broad set of diagnoses.

A 41-year-old man with Child class A alcoholic cirrhosis undergoes his first
screening upper endoscopy. He has no history of upper-gastrointestinal
bleeding. Upper endoscopy identifies the presence of large varices with no
red wheals. Initial primary prophylaxis against variceal hemorrhage for this
patient is
A
observation.
B
beta-blockade.
C
endoscopic variceal sclerotherapy.
D
endoscopic variceal ligation and beta-blockade.
E
spironolactone.

B
beta-blockade.
The initial diagnosis and classification of nonbleeding esophageal varices is
usually made on esophagogastroduodenoscopy.

In most centers, varices are classified by a semiquantitative morphological


assessment into 3 sizes or grades:

1- Small (grade I), generally defined as minimally elevated veins above the
esophageal mucosal surface normal in color, straight, and compressible.

2- Medium (grade II), medium varices defined as tortuous veins occupying


less than one-third of the esophageal lumen, with or without red wheals,
and noncompressible.

3- Large (grade III) defined as those occupying more than one-third of the
esophageal lumen, with or without red wheals, and noncompressible.

Nonselective beta-adrenergic blockers are the foundation of therapy for


primary prophylaxis for preventing variceal bleeding.

Patients with cirrhosis and varices that have not bled should be started on
primary prophylaxis.
A meta-analysis of 11 trials (1189 patients) evaluated nonselective
beta-blockers (e.g., propranolol, nadolol) versus nonactive treatment
or placebo in preventing first variceal hemorrhage.
In patients with large- or medium-sized varices, risk of first variceal
bleeding was significantly reduced by beta-blockers (30% in controls
vs. 14% in patients treated with beta-blockers).
One bleeding episode was avoided for every 10 patients treated.

Two invasive endoscopic approaches to manage esophageal varices are


available:
1- sclerotherapy and
2- banding.
a. Endoscopic sclerotherapy is associated with a higher risk of side
effects compared with variceal band ligation (VBL).
b. Therefore, variceal band ligation is the preferred method for
endoscopic prophylaxis of variceal bleeding.

A meta-analysis of 5 randomized clinical trials comparing VBL with no


treatment showed a decreased risk of first variceal bleeding and a
lower mortality rate in the VBL group.
Spironolactone in combination with beta-blockade shows no benefit over
beta-blockade alone for primary prophylaxis of variceal bleeding.

Nonselective beta-blockers or variceal band ligation may be an appropriate


first-line choice for primary prophylaxis of variceal bleeding.
However, the combination of both therapies is not more effective and is
associated with increased side effects.

A 24-year-old man undergoes an ultrasound of the abdomen for nonspecific


generalized abdominal pain. Three 7- to 8-mm polyps are identified in the
gallbladder. The next step in the management of this finding would be
A
CT scan of the abdomen.
B
endoscopic ultrasound.
C
laparoscopic cholecystectomy.
D
open cholecystectomy with lymph node sampling.
E
repeat ultrasound in 6 months.

E
repeat ultrasound in 6 months.

Gallbladder polyps are identified in 1.5–6.9% of the population.


They can be classified as
- “pseudotumors” (cholesterol polyps, adenomyomas, or inflammatory
polyps),
- epithelial (adenomas),
- mesenchymal (leiomyomas), and
- malignancies (adenocarcinomas).

The main concern when a polyp is identified is whether it is malignant.

Single polyps are more likely to be malignant than multiple polyps.


The feature most predictive of malignancy is the size of the polyp.
- A polyp larger than 1 cm in size has a risk of malignancy of 43–77%,
- whereas polyps larger than 2 cm are nearly always malignant.

Age also needs to be considered;


- patients older than 50–60 years have a higher risk of malignancy.

Finally, the presence of gallstones along with polyps or


- a diagnosis of sclerosing cholangitis and polyps increases the likelihood
of malignancy.

With the knowledge of these risk factors, a care plan algorithm can be
designed.

In a patient with true biliary symptoms, cholecystectomy is indicated based


on the symptoms.
The presence of polyps does not influence that decision.
- In patients with no symptoms and polyps 5 mm or smaller, a follow-up
ultrasound in 6–12 months is indicated.
- If the polyps have not increased in size, no further imaging is needed.
- Polyps 6–9 mm in size can be followed with ultrasound.

Typically, a follow-up study is obtained at 6 months and 1 year and then


yearly thereafter if polyp size is stable.
- If the polyps decrease in size, no further imaging is needed.
- If they increase in size, a cholecystectomy is indicated.

An exception could be a patient over the age of 50 with a single polyp larger
than 5 mm.
In a patient older than 50, cholecystectomy is indicated.

All polyps 10 mm or greater in size require surgery.

CT scanning should be considered in patients with


- polyps greater than 10 mm and age younger than 60 and in
- all patients with polyps greater than 20 mm to evaluate for invasive
cancer.

The role of endoscopic ultrasound in the management of gallbladder polyps


is not well defined.
Studies show it can be used to predict pathology at the time of
cholecystectomy in 97% of patients.
However, it is more invasive and expensive, and it requires endoscopists
who have advanced endoscopic ultrasound training.
Thus, although it may be useful in evaluating and staging larger polyps, it
may not be the most cost-effective way to follow patients with smaller
polyps.
Open, nonradical cholecystectomy with lymph node sampling is not a
procedure used for benign or malignant gallbladder pathology.

A 47-year-old woman presents with fever and left upper quadrant pain. A CT
scan is obtained and shown in figure 1. The most appropriate treatment for
this lesion is

FIGURE 1

A
intravenous antibiotics.
B
intravenous antifungals.
C
percutaneous drainage with intravenous antibiotics.
D
splenectomy.
E
splenic embolization.

C
percutaneous drainage with intravenous antibiotics.

Splenic abscesses are rare, but if not treated appropriately, they have a high
mortality rate (figure 2).

The most common etiology is hematogenous spread, such as from infective


endocarditis.
Splenic trauma is another common etiology, as is splenic infarction.

Up to 50% of these infections are polymicrobial.

For this reason, empiric broad-spectrum antibiotics should be initiated as


soon as the diagnosis is made.

Primary fungal splenic abscesses are rare and antifungal therapy would not
be initiated as part of empiric therapy.
Antibiotics alone are not considered definitive treatment.

Percutaneous drainage is now the procedure of choice for splenic abscess.

Aspiration allows for culture-directed antibiotics in addition to preserving the


spleen.

Contraindications to percutaneous drainage include


- multiple abscesses and cysts that have features that make a diagnosis
of Echinococcus more likely,
o such as a calcified cyst wall and
o the presence of other cysts in the abdominal cavity, particularly
in a patient from the Mediterranean basin or Eastern Europe.
- Additionally, patients who are coagulopathic or
- those without safe percutaneous access to the abscess are not
candidates for percutaneous drainage.

Splenectomy is reserved for patients who are not candidates for or who have
failed percutaneous drainage.

There is no indication for the use of splenic embolization in splenic abscess,


and it is one of the etiologies of splenic abscess.

Two weeks ago, a 69-year-old man with a remote history of right


hemicolectomy for a T2N0 colon cancer was admitted with right upper
quadrant pain, fever, and jaundice. On presentation, he was
hemodynamically abnormal but responsive to fluid resuscitation. Laboratory
values were as follows: white blood cell count of 24,000/mm 3 (3600–
11,200/mm3), bilirubin of 14 mg/dL (0.2–1.9 mg/dL), alkaline phosphatase
of 670 U/L (<95 IU/L), glutamic-oxaloacetic transaminase of 250 U/L (<42
U/L), gamma-glutamyl transferase of 175 U/L (10–70 U/L). Gallbladder
ultrasound showed a thickened gallbladder with a positive Murphy sign but
without cholelithiasis and a normal caliber common bile duct. The patient
initially underwent magnetic resonance cholangiopancreatography
(MRCP; figure 1), followed by a cholecystostomy that drained bile and pus.
Despite broad-spectrum antibiotics for 7 days, his white blood cell count
remains at 15,000/mm3 and his bilirubin is 9 mg/dL. Which of the following
would you recommend?

FIGURE 1

A
Laparoscopic cholecystectomy
B
Liver biopsy
C
Serum IgG4
D
Cancer antigen 19-9 level
E
Endoscopic retrograde cholangiopancreatography with stent placement

C
Serum IgG4
This patient presents a picture of cholangitis without evidence of the distal
bile duct obstruction that would be expected with
- choledocholithiasis,
- a stricture complicating pancreatitis, or
- obstructing pancreatic/common duct tumor.

The incongruity of history/physical and laboratory values should prompt


further investigation before considering operative intervention.
The degree of hyperbilirubinemia and elevated alkaline phosphatase do not
support a diagnosis of acalculous cholecystitis.

Because of its high sensitivity and specificity, noninvasive magnetic


resonance cholangiopancreatography (MRCP) has largely replaced
endoscopic retrograde cholangiopancreatography (ERCP) as the initial
diagnostic test of choice.

The MRCP images for this case show the classic “bead-on-a-string”
appearance of sclerosing cholangitis (figure 2).

Although placement of a stent might be useful for biliary decompression in


the face of a dominant extrahepatic stricture, significant clinical
improvement is unlikely in the presence of intrahepatic strictures.

Because this patient may ultimately be considered for liver transplant,


cholecystostomy drainage and a prolonged course of antibiotics is the most
appropriate initial management for this patient.

Primary sclerosing cholangitis is the classic hepatobiliary manifestation of


inflammatory bowel disease, leading to cirrhosis and end-stage liver disease
caused by chronic and progressive biliary strictures.

Disease with no known precipitant is deemed primary sclerosing cholangitis,


whereas a preceding identifiable biliary injury is designated secondary
disease.
Autoimmune diseases, including IgG4-associated autoimmune pancreatitis,
ischemia, toxins, and inherited disorders, mimic the historical and radiologic
features shown in the MRCP images.
If confirmatory, a trial of corticosteroid immunosuppression is appropriate,
and, in the absence of hepatocellular carcinoma, suitable patients may be
considered for liver transplantation.
Although cancer antigen 19-9 (CA 19-9) is an indicator of pancreatic
carcinoma, it has poor sensitivity for early-stage cholangiocarcinoma.
Because increased CA 19-9 concentrations may result from cholangitis
alone, routine measurement in this setting is less valuable.
Diagnosis of small-duct disease necessitates liver biopsy when a potential
overlap with autoimmune hepatitis is suspected (10% of patients).

Bead on a string appearance

A 48-year-old man with known cirrhosis due to hepatitis C is being screened


for hepatocellular carcinoma. His CT scan shows portal vein thrombosis.
Which of the following would you recommend?
A
Splenectomy
B
Immediate liver transplant
C
Observation
D
Splenorenal shunt
E
Anticoagulation
E
Anticoagulation

Portal vein thrombosis (PVT) is a complication of cirrhosis commonly


identified on imaging of patients with advanced liver disease and is
associated with portal hypertension and hypersplenism.

Chronic partial PVT may be asymptomatic, whereas


acute complete obstruction may induce intestinal congestion with severe
continuous or colicky abdominal pain and occasionally nonbloody diarrhea.

Although PVT and its attendant portal hypertension pose a significant risk of
variceal bleeding, a treatment algorithm combining anticoagulation and
transjugular intrahepatic portosystemic shunting (TIPS) offers the best
chance for
- restoring portal flow,
- reducing portal pressures,
- reducing thrombosis extension, and
- reducing the risk of intestinal infarction. 

Although appropriate for management of bleeding gastric varices due to


splenic vein thrombosis, splenectomy decreases portal vein flow, thus
increasing the risk of PVT.
Splenorenal shunt, an option for management of bleeding varices, also
decreases portal blood flow.

PVT alone is not an indication for liver transplantation nor is it necessarily a


contraindication.

Its presence, however, is an independent risk factor for recurrent PVT after
transplant and decreased perioperative survival.
Complete or partial recanalization is associated with better survival rates
and, therefore, anticoagulation is recommended in all patients with PVT.

A 54-year-old woman presented to her primary care provider with


intermittent, right upper quadrant pain associated with fatty foods.
Ultrasound showed a distended gallbladder without evidence of cholelithiasis.
Blood work included an alkaline phosphatase of 245 U/L (<95 U/L)
associated with a normal bilirubin and serum glutamic-oxaloacetic
transaminase of 28 U/L (10–55 U/L). Based on the magnetic resonance
cholangiopancreatography (MRCP) scan shown (figure 1), which of the
following would you recommend?

[+] CLICK IMAGE TO ENLARGE


FIGURE 1

A
Endoscopic retrograde cholangiopancreatography with biopsy and stent
B
Laparoscopic cholecystectomy with cholangiogram and common duct
exploration
C
Whipple (pancreaticoduodenectomy)
D
Cholecystectomy with bile duct excision and Roux-en-Y hepaticojejunostomy
E
Cholecystectomy with choledochoduodenostomy
D
Cholecystectomy with bile duct excision and Roux-en-Y hepaticojejunostomy
This patient’s workup was initiated by a concern for biliary colic.
The ultrasound did not show cholelithiasis.
Her laboratory picture of normal bilirubin and transaminase values with a
marked elevation in alkaline phosphatase supports further imaging of the
biliary tree.
In this case, the magnetic resonance cholangiopancreatography (MRCP)
shows a type I choledochocyst (figure 2).

There is no evidence for distal or common duct obstruction warranting


endoscopic retrograde cholangiopancreatography (ERCP) with stent or
cholecystectomy with common duct exploration.
A pancreaticoduodenectomy or Whipple procedure is generally reserved for a
distal bile duct or pancreatic head malignancy.

In this case, cholecystectomy with bile duct excision and Roux-en-Y


hepaticojejunostomy is the procedure of choice.

Choledochocysts are single or multiple dilations of the intrahepatic or


extrahepatic biliary tree (figure 3).
The classic triad of jaundice, right upper quadrant pain, and an abdominal
mass is less common in adults than in children.
Untreated, patients are at risk of developing recurrent cholangitis, liver
abscesses, and cholangiocarcinoma.

Those confined to the common bile duct, as in this patient, are well managed
by cholecystectomy with complete cyst excision to include any abnormal
common duct from the hepatic plate to the intrapancreatic portion of the
duct for fusiform dilations and hepaticoenterostomy reconstruction.

The hepaticoenterostomy can be performed as a


- hepaticoduodenostomy or
- a Roux-en-Y hepaticojejunostomy.

The latter approach is preferred, because it excises all potentially


premalignant cyst tissue and separates the biliary tree from the pancreatic
duct, eliminating any mixture of pancreatic and biliary secretions that may
contribute to disease pathogenesis.

Hepaticoduodenostomy carries the risk of bilious gastric reflux, gastritis, and


esophagitis.

Overall complications are substantially lower with Roux-en-Y reconstruction


(7%) compared with hepaticoduodenostomy (42%).
If the cyst cannot be completely excised, stripping or ablation of the mucosa
is recommended and the patient should be followed with regular ultrasound
surveillance for evidence of malignancy development.
A 50-year-old man with alcoholic cirrhosis and ascites presents with
abdominal pain, fever, and leukocytosis. His blood pressure is 88/50 mm Hg,
and his heart rate is 100 beats per minute. Laboratory data are as follows:
Blood urea nitrogen = 45 mg/dL (7–20 mg/dL)
Creatinine = 1.7 mg/dL (0.40–1.30 mg/dL)
International normalized ratio = 2.5 (0.8–1.2)
Albumin = 2.0 g/dL (3.5–5.2 g/dL)
Diagnostic paracentesis demonstrates 500 neutrophils/mm3 of ascitic fluid.
Which of the following therapies is indicated as a first step in treatment?
A
Vancomycin
B
Metronidazole
C
Fresh frozen plasma
D
Cryoprecipitate
E
Third generation cephalosporin
E
Third generation cephalosporin

Spontaneous bacterial peritonitis (SBP) is a bacterial infection of ascitic fluid


without an intra-abdominal source of infection.

SBP is associated with underlying conditions such as cirrhosis, nephrotic


syndrome, and congestive heart failure.

The patient will have abdominal pain, fever, leukocytosis, and an ascitic fluid
sample demonstrating more than 250 neutrophils/mm3.

Broad-spectrum antibiotics, typically a third-generation cephalosporin, are


started immediately.

Gram-positive and anaerobic organisms rarely cause SBP in cirrhotic patients


making vancomycin or metronidazole inappropriate.
Because the patient is not bleeding, neither fresh frozen plasma nor
cryoprecipitate is necessary.

Laparoscopic cholecystectomy carries a higher risk of a bile duct injury than


open cholecystectomy. One approach to mitigate this risk is to achieve the
“critical view of safety.” Which of the following best describes the critical
view of safety?
A
Visualizing contrast in the duodenum on cholangiogram
B
Visualizing 2 tubular structures entering the neck of the gallbladder and the
liver/cystic plate behind
C
Visualizing the cystic duct–common bile duct junction
D
Dissecting the gallbladder free from the liver bed in a top-down fashion
E
Visualizing 2 tubular structures in the triangle of Calot
B
Visualizing 2 tubular structures entering the neck of the gallbladder and the
liver/cystic plate behind

Laparoscopic cholecystectomy is associated with a 0.4% incidence of bile


duct injury.
Errors in identification are the most common cause of bile duct injuries
during laparoscopic cholecystectomy.

The “critical view of safety” is advocated as a means of ensuring the


cystic duct and artery
are clearly identified and that no other structures are adherent to or
mistaken for these structures before dividing them.

The essential elements of this critical view are to


- visualize 2 and only 2 tubular structures entering the neck of the
gallbladder with
- a view of the liver and cystic plate behind these cystic structures
(video 1).

Although intraoperative cholangiography can be complementary to the


critical view of safety, it is not required.
The cystic duct–common bile duct junction should not be routinely dissected,
because this maneuver can actually increase the risk of a bile duct injury.
At times, a top-down mobilization of the gallbladder may be necessary, but
this is not part of or required for the critical view of safety.
Visualizing 2 tubular structures in the triangle of Calot alone does not
constitute this critical view.

A 52-year-old woman with ulcerative colitis underwent a total colectomy


with end ileostomy many years ago. She was subsequently diagnosed with
primary sclerosing cholangitis, and she developed cirrhosis. She now
presents with copious bleeding from her ileostomy with bright red blood and
clots in the bag. She reports intermittent episodes of bright red bleeding into
her ileostomy bag over the past 3 months. She is pale and diaphoretic. Her
heart rate is 130 beats per minute, and her blood pressure is 80/30 mm Hg.
On abdominal examination, she has a caput medusae and moderate ascites
by percussion and bedside ultrasound. After successful resuscitation, she
undergoes an upper endoscopy, which is negative for esophageal varices.
What is the next best step in her management?
A
Transjugular intrahepatic portosystemic shunt
B
Stomal enteroscopy
C
Laparotomy and resection of the terminal ileum
D
Portacaval shunt
E
Beta-blockade
A
Transjugular intrahepatic portosystemic shunt

This patient is most likely bleeding from her peristomal varices given the
negative upper endoscopy.
The patient should be aggressively resuscitated with blood products, and any
underlying coagulopathy should be corrected.
A temporizing measure to consider would be to apply traction to a balloon-
tipped catheter inserted into her ileostomy to temporarily control the
bleeding.

Although endoscopic management is often a first-line therapy in esophageal


variceal bleeding, enteroscopy through the stoma is not likely to be effective
in managing bleeding from peristomal varices, because they arise at the
mucocutaneous junction.

If transjugular intrahepatic portosystemic shunt (TIPS) is available, this


intervention will significantly reduce the risk of current and future bleeding
from peristomal varices.

Operative intervention with portacaval shunt carries a significantly higher


mortality in this case.
Re-siting the ileostomy with or without ileum resection can be considered
but should be performed only if TIPS is not available or other factors indicate
a need for a new ileostomy site.

Nonselective beta-blockers are useful in preventing initial variceal bleeds but


are not indicated for a patient who is actively bleeding and in hemorrhagic
shock.

An 18-year-old Afghan woman presents with upper abdominal fullness and


pain. A chest x-ray (figure 1) and CTs of her chest and abdomen are
obtained (figure 2 and figure 3). What is her diagnosis?

[+] CLICK IMAGE TO ENLARGE


FIGURE 1
[+] CLICK IMAGE TO ENLARGE
FIGURE 2

[+] CLICK IMAGE TO ENLARGE


FIGURE 3

A
Echinococcal cysts
B
Amoebic cysts
C
Polycystic disease
D
Cystadenomas
E
Retained fragments/foreign bodies
A
Echinococcal cysts

Echinococcus is endemic in much of the underdeveloped world, including


Afghanistan.
Four different species of this tapeworm infect humans, with Echinococcus
granulosis being the most common.

The x-ray and CT scan demonstrate classic findings of a calcified cystic wall
in the lungs and liver (figure 4).

Amoebic cysts, polycystic disease, and cystadenomas do not typically have


calcified walls.
Retained fragments or foreign bodies would have more beam-hardening
artifact scatter around the radiodense structures.

This patient underwent successful resection of her multifocal echinococcal


disease in a staged fashion, starting with a left lower lobectomy, followed by
a nonanatomic resection of her right upper lobe and excision of her hepatic
cysts (video 1).
During each procedure, the operative site was surrounded with laparotomy
pads soaked in hypertonic saline, which is toxic to the parasite, and great
care was taken during the cyst excision to avoid spilling the cyst contents.
Spillage of cyst contents is associated with significant morbidity and
mortality.

A 52-year-old man is referred for evaluation of progressive fatigue, 2.7-kg


weight loss, jaundice, and pruritus. His medical history is significant for
ulcerative colitis diagnosed at age 28, which was followed by a total
proctocolectomy with ileoanal pouch anastomosis at age 46. After his
operation, he has done well with 1 episode of “pouchitis” 8 months ago. This
episode resolved promptly with conventional therapy. Physical examination
is negative other than some mild scleral icterus. Laboratory serum
abnormalities include the following: Alkaline phosphatase = 422 IU/L (<95
IU/L), serum glutamic oxaloacetic transaminase = 70 IU/L (<42 IU/L), and
serum alanine transaminase = 65 IU/L (7–55 IU/L). The most likely study to
confirm the diagnosis is
A
cholescintigraphy (HIDA scan).
B
transabdominal ultrasound.
C
magnetic resonance cholangiography.
D
upper gastrointestinal endoscopic ultrasound.
E
serum antinuclear cytoplasmic antibody determination (p-ANCA).
C
magnetic resonance cholangiography.

Fatigue, weight loss, and pruritus are the classic presenting symptoms of
primary sclerosing cholangitis (PSC).
A history of ulcerative colitis, which is present in 75–80% of patients with
PSC, strongly suggests the diagnosis.

Unlike primary biliary cirrhosis (PBC), which is more common in women and
affects the small and medium bile ducts, PSC has a slight male predilection
and can involve both intra- and extrahepatic ducts.

Confirmation of the diagnosis is by cholangiography, preferably magnetic


resonance cholangiography, which has the advantage of being noninvasive
and cost-effective and has a sensitivity of better than 90%. 

Endoscopic retrograde cholangiography, which has a sensitivity approaching


100%, is currently reserved for equivocal finding by magnetic resonance
cholangiography.
Cholescintigraphy (HIDA scan) is not helpful in confirming the diagnosis,
because this imaging modality lacks sufficient detail to show the classic
“beads-on-a-string” anatomic configuration of the ducts.
Percutaneous liver biopsy is not necessary for the diagnosis.
Endoscopic or transabdominal ultrasound, while beneficial in the
identification of choledocholithiasis or pancreatic pathology, is not
recommended for suspected PSC, because it cannot evaluate the
intrahepatic ductal system.

The determination of serum antibodies (e.g., p-ANCA and ANA) is useful in


suggesting autoimmune disorders but lacks the ability to differentiate the
many autoimmune conditions, such as rheumatoid arthritis, Sjögren
syndrome, and celiac disease. 

The elevation of serum alkaline phosphatase is the primary laboratory


abnormality in PSC patients.
Most patients present with normal bilirubin levels; the marked elevation of
bilirubin in this case is suggestive of advanced disease or malignancy.

Patients with a history of ulcerative colitis have an increased risk of


cholangiocarcinoma, and those with a history of “pouchitis” have an even
greater risk.
For patients with PSC, intact colons, and ulcerative colitis, the risk of
colorectal cancer, preferentially the right colon, is increased by some reports
160-fold over normal individuals.
The relationship between PSC and ulcerative colitis remains obscure, and
theories include genetic (variants in HLA genes), geographical (prevalence in
the United States and European countries 100-times greater than in Asia),
bacterial toxins (colonic bacterial overgrowth with exaggerated immune
response), and environmental factors (tobacco smoke and environmental
toxins), or combinations of these.

Endoscopic stenting of the bile ducts, preferentially with self-expanding


metallic stents, has some role in palliation in PSC.
Some studies suggest that stenting of just a few of the stenotic ducts
provides symptomatic relief in most cases.

Sphincterotomy and cholecystectomy have no therapeutic or even palliative


benefit.

Corticosteroids and immunosuppressive agents are not of any value, either


in providing symptomatic relief or in delaying progression of the disease.

However, in patients with symptoms overlapping autoimmune hepatitis,


corticosteroids may have a role.

The only effective therapy for end-stage PSC is liver transplantation.


In fact, PSC is the most common indication for liver transplantation in
Scandinavia and the fifth most common indication in the United States.
Unfortunately, transplanted livers have a 20% chance of developing a
recurrence of PSC, and this may ultimately lead to graft rejection.

A 56-year-old man presents with fatigue, right upper quadrant abdominal


pain and tenderness, and leukocytosis. Amoebic serology is negative. CT
scan of the abdomen with intravenous contrast shows a 5-cm, round, fluid-
filled lesion with an enhancing rim in the right lobe of the liver. Appropriate
treatment includes
A
antibiotics only.
B
antibiotics and percutaneous drainage.
C
needle aspiration.
D
right hepatectomy.
E
percutaneous drainage only.
B
antibiotics and percutaneous drainage.

Liver abscesses can be caused by fungus, bacteria, and amebae.

Patients commonly present with constant, dull right upper quadrant


abdominal pain, probably due to stretching of the liver capsule.
Fever is common.

Abscesses tend to occur in the right lobe, presumably due to the larger
hepatic volume and predominant flow of blood from the superior mesenteric
vein draining the gastrointestinal tract as opposed to the splenic vein.

The presence of multiple abscesses suggests a bacterial or mixed source.

The treatment of liver abscesses has changed with the emergence of medical
technology, particularly imaging and image-guided percutaneous
interventions.
A century ago, patients with multiple hepatic abscesses had a nearly
universal mortality.
Abscesses not amenable to percutaneous drainage may be successfully
treated with antibiotics alone.

In this case, the 5-cm fluid collection in the right lobe would be accessible by
percutaneous means.
The enhancing rim suggests characteristics of a pyogenic abscess.
The negative amoebic serology eliminates amebae as the cause.

Percutaneous drainage, with or without a catheter placement, improves the


likelihood of treatment success.
Occasionally, repeated aspirations are needed and can still result in
treatment success.

The morbidity of surgical drainage or even hepatectomy precludes its use.

In the Western world, bacterial hepatic abscess is most common.


In developing countries, particularly southeast Asia and Africa, amebic
infection is the most frequent cause.
Amebic liver abscess is caused by the protozoa Entamoeba histolytica, and
most can be treated with amebicidal drug therapy alone.

You might also like